Reversing Order of Integration: Evaluating an Intgral

  • Thread starter Thread starter shards5
  • Start date Start date
  • Tags Tags
    Integration
Click For Summary
The discussion focuses on evaluating the integral by reversing the order of integration for the expression ∫ from 0 to 3 ∫ from 9 to y² of y cos(x²) dy dx. The user correctly identifies the new intervals of integration as 0 ≤ y ≤ √x and 0 ≤ x ≤ 9, leading to the transformed integral ∫ from 0 to 9 ∫ from 0 to √x of y cos(x²) dx dy. After performing the first integration, the user encounters difficulty with the resulting integral involving cos(x²) and seeks clarification on the integration process. The conversation highlights the importance of correctly setting integration limits and applying integration techniques, such as integration by parts and substitution. Ultimately, the user resolves the confusion regarding the integration of cos(x²) and confirms the correct approach to the problem.
shards5
Messages
37
Reaction score
0

Homework Statement


Evaluate the integral by reversing the order of integration.
\int^{3}_{0}\int^{9}_{y^2} y cos(x^2) dydx

Homework Equations



...?

The Attempt at a Solution


Drawing the picture out we get a sideways parabola.
From the picture I get the following intervals of integration.
0 \leq y \leq \sqrt{x}
0 \leq x \leq 9
Using the above I get the following integral.
\int^{9}_{0}\int^{sqrt(x)}_{0} y cos(x^2) dxdy
After the first integration I get.
\frac{y^2}{2} cos(x^2)
Plugging in \sqrt{x} and 0 I get the following resulting integral.
\int^{9}_{0} x/2 * cos(x^2) dy
And here is my problem. It has been a while since I took my calculus II so I don't remember how to integrate the above and I am also not sure if I set my intervals of integration correctly.
 
Last edited:
Physics news on Phys.org
Wait a minute...the second last integral is first done wrt y and then the last integral is done wrt x. How did you get cos(x²)? cos(x) would be a constant wrt y. After correction, the last integral will simply become a by parts integral.
 
It was cos(x^2), and I think I got it using integration by substitution. Thanks a lot.
 
Question: A clock's minute hand has length 4 and its hour hand has length 3. What is the distance between the tips at the moment when it is increasing most rapidly?(Putnam Exam Question) Answer: Making assumption that both the hands moves at constant angular velocities, the answer is ## \sqrt{7} .## But don't you think this assumption is somewhat doubtful and wrong?

Similar threads

  • · Replies 105 ·
4
Replies
105
Views
6K
  • · Replies 19 ·
Replies
19
Views
2K
Replies
4
Views
2K
Replies
7
Views
1K
Replies
20
Views
2K
  • · Replies 2 ·
Replies
2
Views
2K
  • · Replies 5 ·
Replies
5
Views
2K
Replies
3
Views
2K
  • · Replies 40 ·
2
Replies
40
Views
4K
  • · Replies 9 ·
Replies
9
Views
2K